Juan has 5 ​ties, 5 ​shirts, and 3 pairs of pants. How many different outfits can he wear if he chooses one​ tie, one​ shirt, and one pair of pants for each​ outfit?

Answers

Answer 1

Answer:48 outfits.

Step-by-step explanation:

Number of outfits that Juan can wear

N =number of ties x number of pants x number of pants

N =4 x 3 x 4

N =48 outfits


Related Questions

slope -1/2, passes through (0, 5)

Answers

Our standard linear slope equation is:

mx + b = y

We're told our slope is 1/2, so that's our m value.

When x = 0 our value is 5. If you need this visualized, then:

1/2 * 0 +b = 5

0 + b = 5

b = 5

So our equation looks like:

x/2 + 5 = y

At the beginning of spring, Samantha planted a small sunflower in her backyard. When it was first planted, the sunflower was 20 inches tall. The sunflower then began to grow at a rate of 1 inch per week. How tall would the sunflower be after 4 weeks? How tall would the sunflower be after w weeks?

Answers

Answer:

24 inches

Step-by-step explanation:

we assume that with a linear relationship between two variables, for any fixed value of x, the observed ________ follows a normal distribution.

Answers

We assume that with a linear relationship between two variables, for any fixed value of x, the observed residuals follows a normal distribution.

This assumption is based on the Central Limit Theorem, which states that when the sample size is large enough, the distribution of sample means will be approximately normal, regardless of the shape of the underlying population distribution.

In the case of a linear relationship between two variables, we can assume that the residuals (the difference between the observed y values and the predicted values based on the linear regression model) follow a normal distribution with mean 0 and constant variance. This assumption is important because it allows us to use statistical methods that rely on normality, such as hypothesis testing and confidence intervals.

Learn more about Normal distribution:

https://brainly.com/question/1846009

#SPJ11

A number times nine + four gives you fifteen if you have time

Answers

Answer:

11 / 9

Step-by-step explanation:

Let number = x

(x * 9) + 4 = 15

9x + 4 = 15

9x = 15 - 4

9x = 11

x = 11 / 9

Hence, the number is 11 /9

James wants to tile his floor using tiles in the shape of a trapezoid to make the pattern a little more interesting he has decided to cut the tiles in half along the median to top base of each time is 12 inches in length and turn bottom base is 16 inches

Answers

Answer: 14 inches

Step-by-step explanation:

Given the following :

James wants to cut a floor tile in the shape of a trapezium with the following dimension :

Top base = 12 inches

Bottom base = 16 inches

To obtain the medan, we take the average of the two bases :

(top base + bottom base) / 2

(12 inches + 16 inches) / 2

28 inches / 2

= 14 inches.

Divide. Express your answer in simplest form.

2 1/2 ÷ 2 1/6

Answers

Answer:

15/13 OR 1.154

Step-by-step explanation:

2 1/2=5/2

2 1/6=13/6

5/2 / 13/6

=5/2 x 6/13

=30/26

=15/13

=1.15384615385

≈ 1.154

How many degrees is the exterior angle of a triangle (w) if the non-adjacent angles (z and y) are 46 and 85?

How many degrees is the exterior angle of a triangle (w) if the non-adjacent angles (z and y) are 46

Answers

Answer:

46

Step-by-step explanation:

WILL GIVE BRAINLIEST IF RIGHT IG....PLEASE get it RIGHT....
find the volume of a cone with a diameter of 12m and a height of 8m.
ANSWER CHOICES
oA. 96m^3
oB. 384 pi m^3
oC. 96 pi m^3
oD. 288 pi m^3

Answers

Answer:

V = 96 pi m^3

Step-by-step explanation:

V = (1/3) pi(r^2)h

Radius = 1/2(Diameter)

R = 1/2(12)

R = 6

Input.  

V = (1/3) pi(6^2)8

Solve.

V = (1/3) pi(36)8

V = (1/3) 904.32

V = 301.44

V = 301.44/pi

V = 96 pi m^3

Answer:

The correct answer is option (C) 96π m³.

Step-by-step explanation:

As per given question we have provided that :

Diameter = 12 mRadius = 12/2 = 6 mHeight = 8 m

Here's the required formula to find the volume of cone :

\({\longrightarrow{\pmb{\sf{V_{(Cone)} = \dfrac{1}{3}\pi{r}^{2}h}}}}\)

➝ V = Volume➝ π = 22/7➝ r = radius➝ h = height

Substituting all the given values in the formula to find the volume of cone :

\({\implies{\sf{V_{(Cone)} = \dfrac{1}{3}\pi{r}^{2}h}}}\)

\({\implies{\sf{V_{(Cone)} = \dfrac{1}{3}\pi{(6)}^{2}8}}}\)

\({\implies{\sf{V_{(Cone)} = \dfrac{1}{3}\pi{(6 \times 6)}8}}}\)

\({\implies{\sf{V_{(Cone)} = \dfrac{1}{3}\pi \times {(36)} \times 8}}}\)

\({\implies{\sf{V_{(Cone)} = \dfrac{1}{3} \times \pi \times 36 \times 8}}}\)

\({\implies{\sf{V_{(Cone)} = \dfrac{1}{\cancel{3}} \times \pi \times \cancel{36} \times 8}}}\)

\({\implies{\sf{V_{(Cone)} = \pi \times 12 \times 8}}}\)

\({\implies{\sf{V_{(Cone)} = \pi \times 96}}}\)

\({\implies{\sf{V_{(Cone)} =96\pi}}}\)

\(\star{\underline{\boxed{\sf{\red{V_{(Cone)} =96\pi \: {m}^{3}}}}}}\)

Hence, the volume of cone is 96π m³.

\(\rule{300}{2.5}\)

You purchase a rectangular piece of metal that has dimensions 5. 0 x 15. 0 x 30. 0 mm and mass 0. 0158 kg. The seller tells you that the metal is gold. To check this, you compute the average density of the piece. What value do you get? were you cheated?.

Answers

The density of metal is 7022.2 kg/m^3 and the metal is not gold. We were cheated.

In the given question, we purchase a rectangular piece of metal that has dimensions 5.0 x 15.0 x 30.0 mm and mass 0.0158 kg.

The seller tells you that the metal is gold.

We have to find the density of piece.

Then Given ,

Mass = 0.0158 kg

Volume = 5.0 mm * 15.0 mm * 30.0 mm

Density = mass / volume

Putting the values

Density = 1.58 * 10^(-2) kg/(5.0*10^{-3}*15.0*10^{-3}*30.0*10^{-3}) m^3

Density = 7022.2 kg/m^3

The density of metal is 7022.2 kg/m^3 and the metal is not gold. We were cheated.

To learn more about density of metal link is here

brainly.com/question/29009519

#SPJ4

pls help on my maths question

pls help on my maths question

Answers

Answer:

Somewhat illegible but looks like the equation should be:

S = u t + 1/2 a t^2

100 = 20 + 1/2 * a * 4    where t = 2

a* 4 = 160

a = 40

PLEASE HELP!! Select all that apply, brainiest for right answer! right answers only please

PLEASE HELP!! Select all that apply, brainiest for right answer! right answers only please

Answers

Answer:

D

Step-by-step explanation:

May someone please help me with this :)

May someone please help me with this :)

Answers

Answer:

a) 80in peri, 320 in²

b) 19.426 peri, 26.139

17. A man stands 4300 feet from the Burj Khalifa in
Dubai (world's tallest building). The angle of
elevation from his eyes (5 feet) to the top of the
building is 32°. How tall is the building?

17. A man stands 4300 feet from the Burj Khalifa inDubai (world's tallest building). The angle ofelevation

Answers

9514 1404 393

Answer:

  b.  2692 ft

Step-by-step explanation:

The relation of interest between the sides of the right triangle is ...

  Tan = Opposite/Adjacent

Using the given numbers, that is ...

  tan(32°) = (building height)/(4300 ft)

Multiplying by 4300 ft, we get ...

  building height = (4300 ft)·tan(32°) ≈ 2686.9 ft

This is the height above the observer's eyes, so the height of the building above the ground is ...

  2687 +5 = 2692 . . . . feet

The building is 2692 feet tall.

A factory sampled hundreds of 9oz bags of chips off of one of their production lines to make sure the bags had the appropriate
amount of chips. They found the amount of chips to be normally distributed with = 9.12 ounces and o= .05 ounces.
PART A
Given this information, fill out the normal distribution chart below for the weight of 9oz bags of chips from this factory.

A factory sampled hundreds of 9oz bags of chips off of one of their production lines to make sure the

Answers

The probability that a bag of chip is lower than 9.07 oz of weight is 0.1587.

What is Normal Distribution ?

A continuous probability distribution for a real-valued random variable is called a normal distribution or a Gaussian distribution.

An example of a continuous probability distribution is the normal distribution, in which the majority of data points cluster in the middle of the range while the remaining ones taper off symmetrically toward either extreme. The distribution's mean is another name for the center of the range.

The normal distribution defines how a variable's values are distributed, just like any probability distribution does. Because it properly depicts the distribution of values for many natural occurrences, it is the most significant probability distribution in statistics.

the mean of the data = 9.12 oz

The SD = 0.05 oz

The probability of the bag to be less than 9.07 oz

x = 9.07

z = x - mean / SD = 9.07 - 9.12/0.05 = - 1

for z = -ve

Ф(z) = 1 - Ф(-z) = 1 - Ф(1) = 1 - 0.8413 = 0.1587

The probability that a bag of chip is lower than 9.07 oz of weight is 0.1587.

To learn more about Normal Distribution refer to :

https://brainly.com/question/20815863

#SPJ1

help meeeeeee.....!!!!
I WILL MARK BRAINIEST!!!

help meeeeeee.....!!!!I WILL MARK BRAINIEST!!!

Answers

You just plug 0 in for either x or y to get the other intercept since it’s in standard form
8x=35
X= 35/8
-7y=35
Y=-5
X-intercept: 35/8
Y intercept: -5

Answer:

X = 35/8

Y = -5

Describe fully the single transformation that maps a onto triangle b

Describe fully the single transformation that maps a onto triangle b

Answers

The single transformation that maps a onto triangle b is a rotation of 180 degrees across the common vertex of triangles a and b

How to determine the transformation?

From the figure, we have the following highlights

Both triangles are congruentBoth triangles have one common vertexBoth triangles meet at a right angle. this implies a 180-degree rotation

The above highlights mean that the single transformation that maps a onto triangle b is a rotation of 180 degrees across the common vertex of triangles a and b

Read more about transformation at:

https://brainly.com/question/4289712

#SPJ1

This is my last question !

This is my last question !

Answers

Answer:

x=50 degrees

Step-by-step explanation:

I'm not sure but hope this helps and have a  wonderful day!!!!

Answer:

x = 48

Step-by-step explanation:

2x + 10 + x + 26 = 180

3x + 36 = 180

3x = 180 - 36

3x = 144

x = 48

Find Sn for the following geometric sequences described.

Find Sn for the following geometric sequences described.

Answers

From the question, the sum of each of the geometric sequence are;

1)  31 3/4

2) 340

3) 11/16

4) -6, 12, -24

What is geometric sequence?

We have that;

Sn = a(1 -\(r^n\))/1 - r

Sn = 16(1 \(- (1/2)^7\))1 - 1/2

Sn = 16(1 - 1/128)/1/2

Sn = 16(127/128) * 2

Sn = 31 3/4

2) Un = a\(r^n\) -1

256 = \(4(4)^n-1\)

64 =\(4^n-1\)

\(4^3 = 4^n-1\)

n = 4

Sn= \(4(4^4 - 1)\)/4 - 1

Sn = 340

3) Since we have a5 then n = 5

Sn = 1(1 - (\(-1/2)^5\))/1 -(-1/2)

Sn = 33/32 * 2/3

= 11/16

4) 30= a(1 -\((-2)^4\))/1 - (-2)

30 = a(-15)/3

30 = -5a

a = 30/-5

a = -6

Then the first three terms are;

-6, 12, -24

Learn more about geometric sequence:https://brainly.com/question/27852674

#SPJ1

on a necklace two-thirds of the bees are blue of the blue beads 1/4 or oval shaped what fraction of the necklace is made up of blue ovals

Answers

on a necklace two-thirds of the bees are blue of the blue beads 1/4 or oval shaped what fraction of the necklace is made up of blue ovals ​

we have that

2/3 ------> blue

1/4(2/3)=2/12-1/6

answer is 1/6

i^32=......................

i^32=......................

Answers

i^32 = (i^4)^8 and i^4 = 1

so (i^4)^8 =1^8 = 1

The trick on all questions like this is to get as many i^4’s out of the way, since they’re all just 1s.

3 over 4 = 1 over 4 m

3 over 4 = 1 over 4 m

Answers

The value of m in the proportion is given as follows m = 3.

What is a proportion?

A proportion is a fraction of a total amount, and the measures are related using a rule of three.

The relations between variables, either direct or inverse proportional, can be built to find the desired measures in the problem.

The proportion is defined as follows:

3/4 = (1/4)m.

As the values have a proportional relationship, the value of m can be obtained applying cross multiplication, as follows:

3/4 = (1/4)m

cross multiplication;

3/4 x 4 = m

m = 3

Hence m =3 is the value that satisfies the proportion in this problem.

More can be learned about proportions at brainly.com/question/24372153

#SPJ1

What is the formula to find P(A) for a series of simple events (ex: tossing a coin and selecting a number at the same time)

Answers

The  probability of event A is P(A) = 1/4 or 0.25.

The formula to find P(A) for a series of simple events is:

P(A) = (number of outcomes that satisfy the event A) / (total number of possible outcomes)

For example, if you are tossing a coin and selecting a number at the same time, and event A is defined as getting a head and an even number, then:

- The number of outcomes that satisfy event A is 1 (getting a head and an even number, i.e., H2)
- The total number of possible outcomes is 4 (H1, H2, T1, T2)
- Therefore, the probability of event A is P(A) = 1/4 or 0.25.

Visit to know more about Probability:-

brainly.com/question/13604758

#SPJ11

determine if each relationship represtents a fuctikn chose yes or no for each relationship​

determine if each relationship represtents a fuctikn chose yes or no for each relationship

Answers

Answer:

Yes

no

no

yes

Step-by-step explanation:

I hope it's serving

:v

Use integration in cylindrical coordinates in order to compute the volume of: U={(x,y,z):0≤x2+y2≤1,0≤z≤5−x−y}

Answers

The volume of the given set using integration in cylindrical coordinates is 12π.

Main part: The given set U can be visualized as a cylinder of radius 1 and height 5, that is truncated by removing a smaller cylinder of the same height and radius 1/2. We will use cylindrical coordinates (ρ,θ,z) to describe points in space. The volume of the set U is given by the integral:

∫∫∫_U〖dV=∫_0^(2π)∫_0^1∫_0^(5-r)ρdzdρdθ

Solution: We can see that z varies from 0 to 5 - x - y. In cylindrical coordinates, x = ρ cos θ and y = ρ sin θ. Therefore, z = 5 - ρ cos θ - ρ sin θ. Since ρ goes from 0 to 1, we have:

∫_0^(2π)∫_0^1∫_0^(5-r)ρdzdρdθ = ∫_0^(2π)∫_0^1∫_0^((5-ρcosθ-ρsinθ))ρdzdρdθ

= ∫_0^(2π)∫_0^1[5ρ - ρ²/2 - ρ² cos θ/2 - ρ² sin θ/2] dρ dθ

= ∫_0^(2π)[5/2 - 1/2 cos θ - 1/2 sin θ] dθ

= 2π [5/2 - 1/2 cos θ - 1/2 sin θ] from 0 to 2π

= π (5 - cos θ - sin θ) from 0 to 2π

= π (5 - (-1) - 0 - (-1) - 0)

= 12 π

Therefore, the volume of the set U is 12 π.

Conclusion: Thus, the volume of the given set using integration in cylindrical coordinates is 12π.

To know more about volume visit

https://brainly.com/question/6071957

#SPJ11

. Find two polynomial expressions whose quotient, when simplified, is 1/x . Use that division problem to determine whether polynomials are closed under division.

Answers

Answer:

The two polynomials are:

(x + 1) and (x² + x)

Step-by-step explanation:

A polynomial is simply an expression which consists of variables & coefficients involving only the operations of addition, subtraction, multiplication, and non - negative integer exponents of variables.

Now, 1 and x are both polynomials. Thus; 1/x is already a quotient of a polynomial.

Now, to get two polynomial expressions whose quotient, when simplified, is 1/x, we will just multiply the numerator and denominator by the same polynomial to get more quotients.

So,

Let's multiply both numerator and denominator by (x + 1) to get;

(x + 1)/(x(x + 1))

This gives; (x + 1)/(x² + x)

Now, 1 and x are both polynomials but the expression "1/x" is not a polynomial but a quotient and thus polynomials are not closed under division.

Which is the approximate measure of angle jgh? 18.4° 19.5° 70.5° 71.6°

Answers

The approximate measure of angle JGH is 70.5°.

According to this question, we have a right triangle in which lengths of its hypotenuse (GH), in centimeters, and one leg (JH) are known and we must determine the measure of an angle (∠JGH), in degrees. A representation of this triangle is included in the image attached below.

By trigonometry we have the following expression for the required angle:

cos∠GJH = HJ/GH               .......(1)

Given, HJ = 2, GH = 6

cos∠GJH = 2/6

     ∠GJH = \(cos^{-1}\frac{2}{6}\)

                ≈ 70.529

The approximate measure of angle JGH is 70.5°.

To learn more about angle here:

https://brainly.com/question/28451077

#SPJ4

Let X
=
A
.
¯¯¯¯¯¯
B
C
. Evaluate X for
(a) A
=
1
,
B
=
0
,
C
=
1
, (b) A = B = C = 1 and ( c) A = B = C = 0.

Answers

The given expressions, when A=1, B=0, and C=1, X evaluates to 1.001; when A=B=C=1, X evaluates to 1.111; and when A=B=C=0, X evaluates to 0.000. These evaluations are based on the given values of A, B, and C, and the notation ¯¯¯¯¯¯BC represents the complement of BC.

To evaluate the expression X = A.¯¯¯¯¯¯BC, we substitute the given values of A, B, and C into the expression.

(a) For A = 1, B = 0, and C = 1:

X = 1.¯¯¯¯¯¯01

To find the complement of BC, we replace B = 0 and C = 1 with their complements:

X = 1.¯¯¯¯¯¯01 = 1.¯¯¯¯¯¯00 = 1.001

(b) For A = B = C = 1:

X = 1.¯¯¯¯¯¯11

Similarly, we find the complement of BC by replacing B = 1 and C = 1 with their complements:

X = 1.¯¯¯¯¯¯11 = 1.¯¯¯¯¯¯00 = 1.111

(c) For A = B = C = 0:

X = 0.¯¯¯¯¯¯00

Again, we find the complement of BC by replacing B = 0 and C = 0 with their complements:

X = 0.¯¯¯¯¯¯00 = 0.¯¯¯¯¯¯11 = 0.000

In conclusion, when A = 1, B = 0, and C = 1, X evaluates to 1.001. When A = B = C = 1, X evaluates to 1.111. And when A = B = C = 0, X evaluates to 0.000. The evaluation of X is based on substituting the given values into the expression A.¯¯¯¯¯¯BC and finding the complement of BC in each case.

Learn more about expressions here:

brainly.com/question/13838432

#SPJ11

what is 8 divided by 7.6

Answers

Answer:

1.05263

explanation:

8 divided by 7.6

\(\hookrightarrow \dfrac{8}{7.6}\)

multiply both by 10

\(\hookrightarrow \dfrac{80}{76}\)

final answer

\(\hookrightarrow 1.05263\)

Please help me, due soon, i will give brainliest!! pre-calc question: according to the intermediate value theorem, in what interval must cos x= 2^x-6 have a solution? show work!

Answers

According to the Intermediate Value Theorem, if a function f(x) is continuous on a closed interval [a, b], and if f(a) and f(b) have opposite signs, then there exists at least one value c in the interval (a, b) such that f(c) = 0. According to the Intermediate Value Theorem, the interval in which the equation cos(x) = 2^(x-6) must have a solution is (0, π/2).

Let's apply this theorem to the function f(x) = cos(x) - 2^(x-6).
First, we need to find two values a and b such that f(a) and f(b) have opposite signs. Let's start with a = 0 and b = π/2.
f(0) = cos(0) - 2^(0-6) = 1 - 1/64 > 0
f(π/2) = cos(π/2) - 2^(π/2-6) = 0 - 1/4 < 0
Since f(a) and f(b) have opposite signs, there exists at least one value c in the interval (0, π/2) such that f(c) = 0. Therefore, the equation cos(x) = 2^(x-6) has at least one solution in the interval (0, π/2).
To summarize, according to the Intermediate Value Theorem, the interval in which the equation cos(x) = 2^(x-6) must have a solution is (0, π/2).

To knnow more about Theorem visit:

https://brainly.com/question/28667736

#SPJ11

There are 100 cm in a meter. write an expression to find the number of centimeters in 8 meters.

Help me with this question plss

Answers

Answer:

800m

Step-by-step explanation:

If 100cm is equivalent to a meter (1 Meter). 8 Meters would be equivalent to 8 Meters multiplied by 100cm and divided by a meter (1 Meter)

Other Questions
Another name for a negotiated labor-management agreement is a? Solve for xx + 8 = 2 How long does an oil change and tire rotation take. Explain why many people consider British Columbias coastal forests a unique habitat that is important to protect. If f(x)= (3+x)/(x-3) what is the f(a+2) In 1557 the Spaniards were able to set up a trading post which was controlled by ______________.a.Spainc.Spain and Chinab.Chinad.The DutchPlease select the best answer from the choices providedABCD the most important mineralcorticoid regulator of electrolyte concentrations in extracellular fluids is . Select the sentence with a verb phrase.A. He likes pizza better than tacos.B. Jude must be playing in his room.C. Mara runs faster than her brother does.D. We finished washing all the dishes. Budgeted sales for the first six months for Meixner Corp. are listed below: APRIL 7,000 JANUARY 6,000 FEBRUARY 7,000 MARCH 8,000 MAY 5,000 JUNE 4,000 UNITS: Meixner Corp. has a policy of maintaining an inventory of finished goods equal to 40 percent of the next month's budgeted sales. If Meixner Corp. plans to produce 6,000 units in June, what are budgeted sales for July? a. 3,600 units 1,000 units c. 9,000 units d. 8,000 units the nurse provides the client wtih a gravity tube feeding via a gastrostomy tube. which action is correct? i need help please and thank you To factor 9x^2-4, you can first rewrite the expression as:A. (3x-2)^2B. (x)^2-(2)^2C. (3x) (2)^2D. None of the above The table below contains some points on the graph of an exponential function. Based on the table, which function represents the same relationship? A group of 4 friends likes to golf together, and each friend keeps track of her all-time lowest score in a single round. Their low scores are all between 90 and 100, except for Angie, whose low score is 80. Angie then golfs a great round and has a new low score of 75. How will decreasing Angie's low score affect the mean and median? During the Texas revolution who commanded the forces that refused to surrender the Alamo a.Sam Houston B.Jim Bowie C.William B.Travis D.Davy Crockett Solve equation 56=-8d+8 the following accounting information pertains to two retail clothing stores. one clothing store has a market strategy of selling only designer brand clothes while the other store sells less expensive bargain brands. tent type bargain brand designer brand sales $ 468,000 $ 404,000 cost of goods sold 416,000 313,100 gross margin 52,000 90,900 average inventory 25,040 55,040 required: calculate the inventory turnover for each clothing store. note: round answers to one decimal place. calculate average days to sell inventory for each clothing store. note: round answers to the nearest whole number. based on your calculations, which clothing store will be required to reorder inventory more frequently? Texas and America have a legislative branch. In this branch there are two different meeting groups. One is called the Senate and one is called the House of Representatives. What vocabulary word is this? It is time to treat your patient. your goal is to return her arterial blood oxygen to normal. use the time and side effect information to drag and drop the treatments to be used first, second and third treatment order treatment options 1. diuretic by injection 1. drop here Raz:Si no sabes no Respondas!